Maximieren eines Winkels basierend auf bestimmten Einschränkungen

A ( 0 , A ) Und B ( 0 , B ) ( A , B > 0 ) sind die Eckpunkte von A B C Wo C ( X , 0 ) ist variabel. Finde den Wert von X wenn Winkel A C B ist maximal.

Jetzt war Geometrie nie wirklich meine Stärke, also entschied ich mich für ein bisschen Kalkül. Zuerst habe ich den Sinussatz verwendet:

S ich N C = B A 2 R
wobei R der Radius des Umkreises ist. Ich stelle fest, dass sinC maximal sein sollte, damit der Winkel C maximal ist. Als solches muss R minimal sein. Als nächstes habe ich die Beziehung verwendet
R = ( B A ) X 2 + B 2 X 2 + A 2 2 Δ
Wo Δ  ist der Bereich von  A B C = ( B A ) X 2 .

Ein bisschen vergleichsweise lange Differenzierung gibt mir den Wert von X als A B .

Wenn ich die Lösungen durchgehe, wird einfach gesagt:

Damit der Winkel ACB maximal ist, berührt der durch A, B verlaufende Kreis die X-Achse bei C.

Darüber hinaus wurde es mit dem sehr einfachen gelöst Ö C 2 = Ö A Ö B , wobei O der Ursprung ist. Die obige Aussage scheint also der Unterschied zwischen einer langwierigen Differenzierung und einer einzeiligen Lösung zu sein.

Es wird ein wenig schwierig für mich zu verstehen, warum die obige Aussage intuitiv sein sollte. Könnte mir jemand etwas mehr Licht ins Dunkel bringen und möglicherweise einen intuitiven Beweis liefern?

Ich konnte mich nicht auf einen prägnanten und dennoch klaren Titel festlegen. Bitte zögern Sie nicht, es zu bearbeiten.

Antworten (6)

Geben Sie hier die Bildbeschreibung ein

Das ist gegeben A , B > 0 also beides A Und B befinden sich auf der gleichen Seite der x-Achse. Der erste Punkt, der zu beachten ist, ist das A B C ist stumpf und A C B ist akut. Jetzt verwenden wir die Beziehung A B = 2 R Sünde C Wo R ist der Umkreis von A B C . Als A B fest ist, maximieren wir C wenn wir minimieren R gegeben Sünde Funktion ist streng zunehmend für ( 0 , π 2 ) .

Beachte das auch Ö muss auf der Mittelsenkrechten von liegen A B die parallel zur x-Achse ist. So, R = Ö C ist minimal, wenn Ö C ist senkrecht zur x-Achse.

„Beides vorausgesetzt A Und B sind über der x-Achse''. Ja, das wurde in der Frage erwähnt. Ich finde es wirklich gut, dass diese Antwort meine Denkweise aufgriff, als ich mich zum ersten Mal mit diesem Problem befasste, und auf diese Weise eine intuitive Erklärung lieferte. Die anderen Antworten, die ich habe, sind auch sehr gut; Es ist schwer, sich auf eine „beste“ Antwort zu einigen.
@C_Lycoris gut zu sehen, dass Sie ein paar Antworten zur Auswahl haben :) Ich war mir nur nicht sicher, ob die Frage das sagte A , B waren beide positiv, also fügte die Linie hinzu, dass sie sich auf der gleichen Seite der x-Achse befinden.
@C_Lycoris Ich sehe das jetzt, also werde ich diese Zeile entfernen
Im Beitrag des OP Ö wird als Ursprung bezeichnet. In Ihrer Antwort?
@ACB beziehe ich mich auf Ö als Zentrum des Umkreises von A B C

Geben Sie hier die Bildbeschreibung ein

Lassen ω sei der Umkreis von A B C Wo C ist ein Punkt auf der X -Achse so dass A C B ist maximal. Annehmen ω schneidet die X -Achse zweimal, bei C Und D .

Lassen F ein beliebiger Punkt auf dem Bogen sein C D (nicht enthalten A , B ) und definieren E als Schnittpunkt von A F Und X -Achse. Beobachten,

A C B = A F B < A E B
was der Tatsache widerspricht, dass A C B ist maximal.

Daher die Annahme, dass ω schneidet die X -axis zweimal ist falsch, was impliziert ω ist eine Tangente an die X -Achse bei C .

+1. Dieser Beweis gefiel mir, er hätte mir eigentlich einfallen sollen.

Lassen K durch A Und B Tangente an die sein X -Achse bei D . Für C auf der X -Achse (und auf der gleichen Seite der j -Achse als D ), lassen A ' Und B ' werden die "anderen" Punkte wo A C Und B C Treffen Sie diesen Kreis.

Geben Sie hier die Bildbeschreibung ein

Eine logische Folge des Satzes über den einbeschriebenen Winkel besagt, dass wir schreiben können

C = 1 2 ( A K B A ' K B ' )
Seit A K B ist fixiert, maximiert C läuft auf Minimierung hinaus A ' K B ' . Dies geschieht, wenn (und nur wenn) A ' Und B ' übereinstimmen; daher wann C Und D übereinstimmen.

Wir nutzen die Tatsache, dass if A Und B auf einem bestimmten Kreis sind, dann, wenn Sie haben C auf dem Kreis u C 0 (ausschließlich) innerhalb des Kreises (und C , C 0 befinden sich auf der gleichen Seite A B ¯ )

A C 0 B > A C B

Sie können dies sehen, indem Sie erweitern A C 0 zum Kreis um C 0 ' , in welchem ​​Fall A C 0 B > A C 0 ' B , aber wegen des einbeschriebenen Winkelsatzes A C 0 ' B = A C B .

Schreiben Sie nun den gewünschten Winkel auf A C B als F ( X ) bezüglich X ; wir wollen maximieren F ( X ) .

Der Umkreis von A B C schneidet immer die X -Achse bei C = ( X 0 , 0 ) . Sagen Sie jetzt zum Widerspruch das F ( X 0 ) maximal ist, und dass der Umkreis von A B C schneidet auch die X -Achse bei C ' = ( X 0 ' , 0 ) C . Dann der Mittelpunkt M von C C ' ¯ ist innerhalb des Kreises, also A M B > A C B ; Und M = ( X 0 + X 0 ' 2 , 0 ) ist auf X -Achse, also

F ( X 0 + X 0 ' 2 ) > F ( X 0 ) ,
widerspricht der Tatsache, dass F ( X 0 ) ist maximal.

Also der Umkreis von A B C muss die schneiden X -Achse an genau einem Punkt.

Es gibt zwei Betrachtungsweisen.

Kreise wie solche, die durch nicht rote Seiten gehen, die sich auf der x-Achse bei D, E treffen, bilden denselben Winkel von Segment AB (Winkel in einem Segment sind gleich). Damit es einen eindeutigen Punkt gibt, sollten diese Punkte zusammengezogen werden, um sie zu einem wiederholten Punkt zu machen. Ein wiederholter Punkt ist tatsächlich ein Tangentialpunkt bei C.

Durch die Eigenschaft des Kreises, dass das Produkt der Segmente konstant ist (dies ist die Kraft des Kreises ), haben wir

(1) Ö A Ö B = Ö C 2 = X 2 X = A B

Geben Sie hier die Bildbeschreibung ein

Der nächste Weg ist die direkte Bestätigung mit Differentialrechnung, Maxima/Minima.

Der "Blickwinkel" oder unterlegte Winkel ist

bräunen 1 A X bräunen 1 B X

Differenzieren bzgl X Arctan und Kettenregel

A / X 2 1 + A 2 / X 2 + B / X 2 1 + B 2 / X 2 = 0

Vereinfacht erhalten wir das gleiche Ergebnis wie (1).

Eine andere Möglichkeit, es mit Analysis und Geometrie zu lösen, besteht darin, dies zu bemerken C = π 2 ( A C Ö + C B Ö ) (Wo Ö ist der Ursprung). Minimierung A C Ö + C B Ö ist gleichbedeutend mit Minimierung bräunen A C Ö + bräunen C B Ö = A X + X B was leicht zu unterscheiden ist.

Es tut mir leid, aber worauf bezieht sich Punkt 'O' in Ihrer Antwort? Wenn es der Ursprung ist, sollte nicht C = π 2 ( A C Ö + C B Ö ) ?
Danke! Ich hatte einen Tippfehler, aber die Funktion, die die Summe der Tangenten ausdrückt, ist korrekt. Punkt Ö ist der Ursprung.
Ja. Ich sah, dass der letzte Ausdruck richtig war. Auch eine sehr schnelle Differenzierung, um die Antwort zu liefern. Danke schön!